MATSEC

Pure Mathematics

Past Paper Solutions











  • Welcome to:



    How to use the Catalogue


    Go the filter bar on the top and choose the solutions you want to access by Year, Session (May or Sept), Paper (1 or 2) and Topic. For example in the following we are choosing the questions of Paper 1 that are related to the topic of Vectors, for all years and both May and September sessions.



    The list of questions that satisfy the filters set above, are shown on the left of the screen. For example.



    Now choose a question from the list on the left.



    The solution of the chosen question will then be displayed in the middle of you screen. For example:


  • September 2019 Paper 1 Question 1

    We would like to solve:
    (1+\cos(2x)\frac{dy}{dx}=2(2y+1)\sin(2x)

    Separating y and x on the opposite sides of the equation:

    \begin{aligned} \frac{1}{2y+1}dy&=\frac{2\sin(2x)}{1+\cos(2x)}dx\\ \int\frac{1}{2y+1}dy&=\int\frac{2\sin(2x)}{1+\cos(2x)}dx\ \ \ \ ...(1) \end{aligned}

    Integrating the right side of Equation (1) by substitution:

    \text{Let }I=\int\frac{2\sin(2x)}{1+\cos(2x)}dx
    \begin{aligned} \text{Let }u&=1+\cos(2x)\\ \frac{du}{dx}&=-2\sin(2x)\\ du&=-2\sin(2x)dx \end{aligned}

    \begin{aligned} I &=-\int\frac{1}{u} du\\ &=-\ln\lvert u\rvert\\ &=-\ln\lvert 1+\cos(2x)\rvert \end{aligned}

    Performing integration on both sides of Equation (1):

    \frac{1}{2}\ln\lvert 2y+1\rvert=-\ln\lvert 1+\cos(2x)\rvert+C

    Using the trigonometric identity \cos2A=2\cos^2 A-1:

    \begin{aligned} \frac{1}{2}\ln\lvert 2y+1\rvert&=-\ln\lvert 2\cos^2 x\rvert+C\\ \ln\lvert 2y+1\rvert^{\frac{1}{2}}&=\ln\lvert 2\cos^2 x\rvert^{-1}+C \end{aligned}

    Since the input of a square root function is positive and 2\cos^2 x is positive:

    \begin{aligned} \ln( 2y+1)^{\frac{1}{2}}&=\ln( 2\cos^2 x)^{-1}+C\\ e^{\ln(2y+1)^{\frac{1}{2}}}&=e^{\ln( 2\cos^2 x)^{-1}+C}\\ e^{\ln(2y+1)^{\frac{1}{2}}}&=e^{\ln( 2\cos^2 x)^{-1}}e^{C} \end{aligned}

    Letting A=e^C:
    (2y+1)^{\frac{1}{2}}=A( 2\cos^2 x)^{-1}

    The General Solution is:
    ( 2y+1)^{\frac{1}{2}}= \frac{1}{2}A\sec^2 x\ \ \ \ ...(2)

    Let x=0 and y=0 in Equation (2):
    \begin{aligned} ( 2(0)+1)^{\frac{1}{2}}&= \frac{1}{2}A\sec^2 (0)\\ A&=2 \end{aligned}

    Substituting A=2 in Equation (2), we get the Particular Solution:

    ( 2y+1)^{\frac{1}{2}}= \sec^2 x
  • September 2019 Paper 1 Question 2

    (a) Consider the curve:
    x^2-xy+y^2=12\ \ \ \ ...(1)

    Differentiating all terms by x:

    \begin{aligned} \frac{d(x^2)}{dx}-\frac{d(xy)}{dx}+\frac{d(y^2)}{dx}&=\frac{d(12)}{dx}\\ 2x-(x\frac{dy}{dx}+y)+2y\frac{dy}{dx}&=0\\ 2x-x\frac{dy}{dx}-y+2y\frac{dy}{dx}&=0\\ (2y-x)\frac{dy}{dx}&=y-2x\\ \frac{dy}{dx}&=\frac{y-2x}{2y-x}\ \ \ \ ...(2) \end{aligned}

    To find the stationary points, let \frac{dy}{dx}=0 in Equation (2). Therefore:

    \begin{aligned} \frac{y-2x}{2y-x}&=0\\ y-2x&=0\\ y&=2x \end{aligned}

    To find the x-values of the stationary points, substitute y=2x in Equation (1):

    \begin{aligned} x^2-xy+y^2&=12\\ x^2-x(2x)+(2x)^2&=12\\ x^2-2x^2+4x^2&=12\\ 3x^2&=12\\ x^2&=4\\ x&=\pm 2. \end{aligned}

    To find the y-values of the stationary points, substitute x=\pm 2 in y=2x:

    When x=-2,y=2(-2)=-4.

    When x=2,y=2(2)=4.

    Hence the coordinates of the stationary points are (-2,-4) and (2,4).

    (b) Let us find \frac{d^2y}{dx^2} by differentiating Equation (2) (found in Part (a)) using the quotient rule.

    \begin{aligned} u&=y-2x\\ u'&=\frac{dy}{dx}-2 \end{aligned} \begin{aligned} \qquad \end{aligned} \begin{aligned} v&=2y-x\\ v'&=2\frac{dy}{dx}-1 \end{aligned}

    By the quotient rule:

    \begin{aligned} \frac{d^2y}{dx^2}&=\frac{vu'-uv'}{v^2}\\ &=\frac{(2y-x)(\frac{dy}{dx}-2)-(y-2x)(2\frac{dy}{dx}-1)}{(2y-x)^2}\\ &=\frac{2y\frac{dy}{dx}-x\frac{dy}{dx}-4y+2x-2y\frac{dy}{dx}+4x\frac{dy}{dx}+y-2x}{(2y-x)^2}\\ &=\frac{3x\frac{dy}{dx}-3y}{(2y-x)^2} \end{aligned}

    At the stationary points, \frac{dy}{dx}=0. Hence:

    \begin{aligned} \frac{d^2y}{dx^2}&=-\frac{3y}{(2y-x)^2}\\ &=-\frac{3y}{(x-2y)^2}\\ (x-2y)\frac{d^2y}{dx^2}&=-\frac{3y}{(x-2y)} \end{aligned}

    Comparing this formula with (x-2y)\frac{dy^2}{dx^2}=k, we get:

    k=-\frac{3y}{x-2y}

    At the stationary point (-2,-4): k=-\frac{3(-4)}{-2-2(-4)}=2

    At the stationary point (2,4): k=-\frac{3(4)}{2-2(4)}=2

    Hence, k=2.

    (c) Consider the second order derivative at the stationary points \frac{d^2y}{dx^2}=-\frac{3y}{(2y-x)^2} as found in Part (b).

    At (-2,-4):

    \begin{aligned} \frac{d^2y}{dx^2}&=-\frac{3(-4)}{(2(-4)-(-2))^2}\\ &=\frac{12}{36}\\ &>0 \end{aligned}

    Hence (-2,-4) is a minimum point.

    At (2,4):

    \begin{aligned} \frac{d^2y}{dx^2}&=-\frac{3(4)}{(2(4)-2)^2}\\ &=-3\\ &\lt 0 \end{aligned}


    Hence (2,4) is a maximum point.
  • September 2019 Paper 1 Question 3

    (a) The equations of the two lines l_1 and l_2 can be written as:

    \begin{aligned} \bf{r}&=\begin{pmatrix}-5\\1\\10\end{pmatrix}+\lambda\begin{pmatrix}-3\\0\\4\end{pmatrix}\\ &=\begin{pmatrix}-5-3\lambda\\1\\10+4\lambda\end{pmatrix} \end{aligned} \begin{aligned} \qquad\qquad \end{aligned} \begin{aligned} \bf{r}&=\begin{pmatrix}3\\0\\-9\end{pmatrix}+\mu\begin{pmatrix}1\\1\\7\end{pmatrix}\\ &=\begin{pmatrix}3+\mu\\\mu\\-9+7\mu\end{pmatrix} \end{aligned}


    To find the point of intersection P we equate the two lines:

    \begin{pmatrix}-5-3\lambda\\1\\10+4\lambda\end{pmatrix}=\begin{pmatrix}3+\mu\\\mu\\-9+7\mu\end{pmatrix}


    The 1st, 2nd and 3rd row give the following three equations:

    \begin{aligned} -5-3\lambda=3+\mu \end{aligned} \begin{aligned} \qquad \end{aligned} \begin{aligned} 1=\mu \end{aligned} \begin{aligned} \qquad \end{aligned} \begin{aligned} 10+4\lambda=-9+7\mu \end{aligned}

    The 2nd equation gives us immediately that \mu=1. Let \mu=1 in the equation of line l_2 to obtain the coordinates of P:
    \begin{aligned} \bf{r}&=\begin{pmatrix}3+1\\1\\-9+7(1)\end{pmatrix}\\ &=\begin{pmatrix}4\\1\\2\end{pmatrix} \end{aligned}
    Hence: P\begin{pmatrix}4\\1\\2\end{pmatrix}.

    (b) We need to show that there exists a real number \mu that satisfies the equation:

    \begin{pmatrix}5\\2\\5\end{pmatrix}=\begin{pmatrix}3\\0\\-9\end{pmatrix}+\mu\begin{pmatrix}1\\1\\7\end{pmatrix}

    Solving for \mu:
    \begin{aligned} \begin{pmatrix}2\\2\\14\end{pmatrix}&=\mu\begin{pmatrix}1\\1\\7\end{pmatrix}\\ \therefore \mu&=2 \end{aligned}

    Since \mu exists, Q\begin{pmatrix}5\\2\\5\end{pmatrix} lies on the line l_2. \square

    (c) Let M \begin{pmatrix}x_1\\y_1\\z_1\end{pmatrix}.

    Consider the vector \vec{MQ}, by the triangle law:

    \begin{aligned} \vec{MQ}&=\vec{OQ}-\vec{OM}\\ &=\begin{pmatrix}5-x_1\\2-y_1\\5-z_1\end{pmatrix} \end{aligned}

    \vec{MQ} is perpendicular to the line l_1. Hence \vec{MQ} is perpendicular to \begin{pmatrix}-3\\0\\4\end{pmatrix} (the directional vector of l_1). Hence:
    \begin{aligned} \begin{pmatrix}-3\\0\\4\end{pmatrix}\cdot\begin{pmatrix}5-x_1\\2-y_1\\5-z_1\end{pmatrix}&=0\\ -3(5-x_1)+4(5-z_1)&=0\\ 3x_1-4z_1&=-5\ \ \ \ ...(1) \end{aligned}

    On the other hand, since M lies on the line l_1:

    \begin{aligned} \begin{pmatrix}x_1\\y_1\\z_1\end{pmatrix}&=\begin{pmatrix}-5\\1\\10\end{pmatrix}+\lambda\begin{pmatrix}-3\\0\\4\end{pmatrix}\\ &=\begin{pmatrix}-5-3\lambda\\1\\10+4\lambda\end{pmatrix} \end{aligned}

    This gives us the following 3 equations:

    \begin{aligned} x_1=-5-3\lambda\ \ \ \ ...(2) \end{aligned} \begin{aligned} \qquad \end{aligned} \begin{aligned} y_1=1\ \ \ \ ...(3) \end{aligned} \begin{aligned} \qquad \end{aligned} \begin{aligned} z_1=10+4\lambda\ \ \ \ ...(4) \end{aligned}

    Equation (2) gives us immediately that y_1=1. Now let us solve Equations (2) and (4) simulataneously to get rid of \lambda. Make \lambda subject of the formula in Equation (4) and insert it in Equation (2):
    \begin{aligned} z_1&=10+4\lambda\\ \therefore\lambda&=\frac{z_1-10}{4} \end{aligned} \begin{aligned} \qquad \end{aligned} \begin{aligned} x_1&=-5-3\lambda\\ x_1&=-5-3(\frac{z_1-10}{4})\\ \therefore x_1&=-\frac{3}{4}z_1+\frac{10}{4}\ \ \ \ ...(5) \end{aligned}

    Let us solve Equations (1) and (5) simulataneously to obtain values of x_1 and z_1. Inserting Equation (5) in Equation (1) gives:

    \begin{aligned} 3x_1-4z_1&=-5\\ 3(-\frac{3}{4}z_1+\frac{10}{4})-4z_1&=-5\\ \therefore z_1&=2\\ \end{aligned}

    Inserting z_1=2 in Equation (5) gives:
    \begin{aligned} x_1&=-\frac{3}{4}(2)+\frac{10}{4}\\ \therefore x_1&=1 \end{aligned}

    Hence M=\begin{pmatrix}1\\1\\2\end{pmatrix}.

    (d) Consider the right-angled triangle \triangle PQM.

    \begin{aligned} \lvert\vec{MP}\rvert&=\lvert \vec{OP}-\vec{OM}\rvert \text{ (by the triangle law)}\\ &=\begin{vmatrix} \begin{pmatrix}4\\1\\-2\end{pmatrix}-\begin{pmatrix}1\\1\\2\end{pmatrix}\end{vmatrix}\\ &=\begin{vmatrix}\begin{pmatrix}3\\0\\-4\end{pmatrix}\end{vmatrix}\\ &=\sqrt{9+0+16}\\ &=5 \end{aligned}

    \begin{aligned} \lvert\vec{MQ}\rvert&=\lvert \vec{OQ}-\vec{OM}\rvert \text{ (by the triangle law)}\\ &=\begin{vmatrix} \begin{pmatrix}5\\2\\5\end{pmatrix}-\begin{pmatrix}1\\1\\2\end{pmatrix}\end{vmatrix}\\ &=\begin{vmatrix}\begin{pmatrix}4\\1\\3\end{pmatrix}\end{vmatrix}\\ &=\sqrt{16+1+9}\\ &=\sqrt{26} \end{aligned}

    \begin{aligned} \therefore\text{Area of }\triangle PMQ&=\frac{\text{length}\times \text{breadth}}{2}\\ &=\frac{5\sqrt{26}}{2}\\ &=\frac{5}{2}\sqrt{26} \end{aligned}
  • September 2019 Paper 1 Question 4

    * This solution involves 3 topics: Remainder & Factor Theorem, Partial Fractions, and the Binomial Theorem

    (a) Consider the function f(x)=4x^3+4x^2-7x+2.

    Let x=-2:
    \begin{aligned} f(-2)&=4(-2)^3+4(-2)^2-7(-2)+2\\ &=4(-8)+4(4)+14+2\\ &=0 \end{aligned}

    \therefore\ x=-2 is a root of 4x^3+4x^2-7x+2.
    \therefore\ (x+2) is a factor of 4x^3+4x^2-7x+2.

    Now let us factorise f(x) completely. Let:

    4x^3+4x^2-7x+2\equiv (x+2)(ax^2+bx+c)\ \ \ \ ...(1)

    Expanding the RHS of the identity:

    4x^3+4x^2-7x+2\equiv ax^3+(2a+b)x^2+(c+2b)x+2c

    Equating coefficients of x^3:
    4=a
    Equating coefficients of x^2:
    \begin{aligned} 4&=2a+b\\ 4&=2(4)+b\text{ since }a=4\\ b&=-4 \end{aligned}

    Equating coefficients of x:
    \begin{aligned} -7&=c+2b\\ -7&=c+2(-4)\text{ since }b=-4\\ c&=1 \end{aligned}

    Hence Equation (1) becomes:

    \begin{aligned} 4x^3+4x^2-7x+2&\equiv (x+2)(4x^2-4x+1)\\ &\equiv (x+2)(2x-1)(2x-1)\\ &\equiv (x+2)(2x-1)^2 \end{aligned}


    (b) (i)
    \begin{aligned} f(x)=&\frac{10-17x+14x^2}{4x^3+4x^2-7x+2}\\ =&\frac{10-17x+14x^2}{(x+2)(2x-1)^2}\text{ (by Part (a))} \end{aligned}

    This is the case where we have repeated linear factors in the denominator. Hence the function is split in partial fractions as follows:

    \frac{10-17x+14x^2}{(x+2)(2x-1)^2}\equiv \frac{A}{x+2}+\frac{B}{(2x-1)}+\frac{C}{(2x-1)^2}

    Finding a common denominator for the LHS:

    \begin{aligned} \frac{10-17x+14x^2}{(x+2)(2x-1)^2}&\equiv \frac{A(2x-1)^2+B(x+2)(2x-1)+C(x+2)}{(x+2)(2x-1)^2}\\ \therefore 10-17x+14x^2&\equiv A(2x-1)^2+B(x+2)(2x-1)+C(x+2)\ \ \ \ ...(2) \end{aligned}

    Let x=\frac{1}{2} in Equation (2):
    \begin{aligned} 10-17(\frac{1}{2})+14(\frac{1}{2})^2&=C(\frac{1}{2}+2)\\ \therefore C&=2 \end{aligned}

    Let x=-2 in Equation (2):

    \begin{aligned} 10-17(-2)+14(-2)^2&=A(-4-1)^2\\ \therefore A&=4 \end{aligned}
    Let x=0 in Equation (2):
    \begin{aligned} 10&=A(1)+B(2)(-1)+C(2)\\ 10&=A-2B+2C\\ 10&=4-2B+2(2)\text{ (since }A=4\text{ and }C=2)\\ \therefore B&=-1 \end{aligned}

    Therefore:
    f(x)=\frac{4}{x+2}-\frac{1}{(2x-1)}+\frac{2}{(2x-1)^2}


    (b) (ii) Let us obtain each fraction of f(x) in a form that could be expanded by the Binomial Theorem.

    \begin{aligned} f(x)&=\frac{4}{x+2}-\frac{1}{(2x-1)}+\frac{2}{(2x-1)^2}\\ &=4(x+2)^{-1}-(2x-1)^{-1}+2(2x-1)^{-2}\\ &=4(2+x)^{-1}-(-1+2x)^{-1}+2(-1+2x)^{-2}\\ &=4[2(1+\frac{x}{2})]^{-1}-[-1(1-2x)]^{-1}+2[-1(1-2x)]^{-2}\\ &=4(\frac{1}{2})(1+\frac{x}{2})^{-1}-\frac{1}{(-1)}(1-2x)^{-1}+2\frac{1}{(-1)^2}(1-2x)^{-2}\\ &=2(1+\frac{x}{2})^{-1}+(1-2x)^{-1}+2(1-2x)^{-2} \end{aligned}

    Now expanding each of these three terms using the Binomial Theorem gives:

    \begin{aligned} f(x)&=2[1+(-1)(\frac{x}{2})+\frac{(-1)(-2)}{1\cdot2}(\frac{x}{2})^2+\frac{(-1)(-2)(-3)}{1\cdot 2 \cdot 3}(\frac{x}{2})^3+\cdots]\\ &\ \ \ +[1+(-1)(-2x)+\frac{(-1)(-2)}{1\cdot 2}(-2x)^2+\frac{(-1)(-2)(-3)}{1\cdot 2 \cdot 3}(-2x)^3+\cdots]\\ &\ \ \ +2[1+(-2)(-2x)+\frac{(-2)(-3)}{1\cdot 2}(-2x)^2+\frac{(-2)(-3)(-4)}{1\cdot 2 \cdot 3}(-2x)^3+\cdots]\\ &=2[1-\frac{1}{2}x+\frac{x^2}{4}-\frac{x^3}{8}+\cdots]\\ &\ \ \ +[1+2x+4x^2+8x^3+\cdots]\\ &\ \ \ +2[1+4x+12x^2+32x^3+\cdots]\\ &=5+9x+28\frac{1}{2}x^2+71\frac{3}{4}x^3+\cdots \end{aligned}

    Let us find the range of values for which this expansion is valid. The following two inequalities must be satisfied.
    \begin{aligned} -1\lt\frac{x}{2}\lt 1\\ -2\lt x \lt 2 \end{aligned} \begin{aligned} \qquad\qquad \end{aligned} \begin{aligned} -1\lt -2x \lt 1\\ -\frac{1}{2}\lt x \lt \frac{1}{2} \end{aligned}

    The intersection of these two intervals is: -\frac{1}{2}\lt x \lt \frac{1}{2}
  • September 2019 Paper 1 Question 5

    Solution Matrix Trans
  • September 2019 Paper 1 Question 6

    Xi curve sketching
  • September 2019 Paper 1 Question 7

    (a) Let I=\int(3x^2-x+1)\sin{x}dx
    \begin{aligned} u&=3x^2-x+1\\ u'&=6x-1 \end{aligned} \begin{aligned} \qquad \end{aligned} \begin{aligned} v&=\int \sin{x} dx\\ v&=-\cos{x} \end{aligned}

    Integrating by parts:
    \begin{aligned} I&=uv-\int vu'dx\\ &=(3x^2-x+1)(-\cos{x})-\int(-\cos{x})(6x-1)dx\\ &=(-3x^2+x-1)\cos{x}+\int(6x-1)\cos{x}dx \end{aligned}

    Let J=\int(6x-1)\cos{x}dx
    \begin{aligned} u&=6x-1\\ u'&=6 \end{aligned} \begin{aligned} \qquad \end{aligned} \begin{aligned} v&=\int \cos{x} dx\\ v&=\sin{x} \end{aligned}

    Integrating by parts:
    \begin{aligned} J&=uv-\int vu'dx\\ &=(6x-1)\sin{x}-\int 6\sin{x}dx\\ &=(6x-1)\sin{x}+6\cos{x}+C \end{aligned}

    \begin{aligned} \therefore I&=(-3x^2+x-1)\cos{x}+J\\ &=(-3x^2+x-1)\cos{x}+(6x-1)\sin{x}+6\cos{x}+C\\ &=(-3x^2+x+5)\cos{x}+(6x-1)\sin{x}+C \end{aligned}


    (b) (i) Let y=a^x.
    \begin{aligned} y&=a^x\\ &=(e^{\ln{a}})^x\\ &=e^{(\ln{a})x}\text{ (using the Laws of Indices)} \end{aligned}

    \begin{aligned} \therefore\frac{dy}{dx}&=e^{(\ln{a})x}\ln{a}\text{ (by the Chain Rule)}\\ &=(e^{\ln{a}})^x\ln{a}\text{ (using the Laws of Indices)}\\ &=a^x\ln{a}\ \square \end{aligned}


    (b) (ii) Let I=\int 2^x\sqrt{2^x+1}dx and let us use the substitution u=2^x:
    \begin{aligned} u&=2^x\ \ \ \ ...(1)\\ \frac{du}{dx}&=2^x\ln{2}\\ \frac{du}{\ln{2}}&=2^x dx\ \ \ \ ...(2) \end{aligned}

    \begin{aligned} I&=\int 2^x\sqrt{2^x+1}dx\\ &=\int \frac{\sqrt{u+1}du}{\ln{2}}\text{ (substituting Equations (1) and (2))}\\ &=\frac{1}{\ln{2}}\int (u+1)^{\frac{1}{2}}du\\ &=\frac{2(u+1)^{\frac{3}{2}}}{3\ln{2}}+C\\ &=\frac{2(2^x+1)^{\frac{3}{2}}}{3\ln{2}}+C \end{aligned}

    \begin{aligned} &\therefore\int_0^3 2^x\sqrt{2^x+1}dx\\ &=\begin{bmatrix}\frac{2(2^x+1)^{\frac{3}{2}}}{3\ln{2}}\end{bmatrix}_0^3\\ &=\frac{2(2^3+1)^{\frac{3}{2}}}{3\ln{2}}-\frac{2(2^0+1)^{\frac{3}{2}}}{3\ln{2}}\\ &=\frac{2(27-\sqrt{8})}{3\ln{2}} \end{aligned}
  • September 2019 Paper 1 Question 9

    (a) Let us count the number of 6-digit positive integers.


    The 1st digit can be chosen in 9 ways. Any value from 1,2,3,4,5,6,7,8,9 is possible. Note that 0 is not included because otherwise we would obtain an integer having 5 or less digits.

    Each of the 2nd, 3rd, \cdots , 6th digits can be chosen in 10 ways. Any value from 0,1,2,3,4,5,6,7,8,9 is possible.

    By the multiplication principle, the number of 6-digit positive integers is:

    9\times10\times10\times10\times10\times10=900000

    Now let us count the number of 6-digit positive integers that begin and end with an even digit.


    The 1st digit can be chosen in 4 ways. Any value from 2,4,6,8 is possible. Again 0 is not included because otherwise we would obtain an integer having 5 or less digits.

    The 6th digit can be chosen in 5 ways. Any value from 0,2,4,6,8 is possible.

    Each of the 2nd, 3rd, 4th, 5th digits can be chosen in 10 ways. Any value from 0,1,2,3,4,5,6,7,8,9 is possible.

    By the multiplication principle, the number of 6-digit positive integers beginning and ending in an even digit is:

    4\times10\times10\times10\times10\times5=200000

    \begin{aligned}\therefore\text{Probability}&=\frac{200000}{900000}\\&=\frac{2}{9}\end{aligned}

    (b) (i) The set of 6-digit positive integers containing the digit 7 once can be partitioned into the following sets:

    ➀ The set of 6-digit positive integers containing the digit 7 only in the 1st place
    ➁ The set of 6-digit positive integers containing the digit 7 only in the 2nd place
    ➂ The set of 6-digit positive integers containing the digit 7 only in the 3rd place
    \qquad\qquad\vdots
    ➅ The set of 6-digit positive integers containing the digit 7 only in the 6th place

    ➀ Let us count the number of 6-digit positive integers containing the digit 7 only in the 1st place.


    The 1st digit can be chosen in just 1 way, by choosing it to be 7.

    The remaining digits can be chosen in 9 ways, that is, any digit except 7.

    By the multiplication principle, the number of 6-digit positive integers containing the digit 7 only in the 1st place is:

    1\times9\times9\times9\times9\times9=59049

    ➁ Let us count the number of 6-digit positive integers containing the digit 7 only in the 2nd place.


    The 1st digit can be chosen in 8 ways, by choosing any digit from 1,2,3,4,5,6,8,9

    The 2nd digit can be chosen in just 1 way, by choosing it to be 7.

    The remaining digits can be chosen in 9 ways, that is, any digit except 7.

    By the multiplication principle, the number of 6-digit positive integers containing the digit 7 only in the 2nd place is:

    8\times1\times9\times9\times9\times9=52488

    ➂ - ➅ The cases for the 6-digit positive integers containing 7 only in the 3rd, 4th, 5th and 6th place are similar to previous case.

    Therefore the number of 6-digit positive integers containing the digit 7 once is:

    59049+5(52488)=321489


    (b) (ii) The set of 6-digit positive integers containing the digit 7 and 8 only once can be partitioned into the following sets:

    ➀ The set of 6-digit positive integers containing the digit 7 and 8 only once and either 7 or 8 is in the 1st place.
    ➁ The set of 6-digit positive integers containing the digit 7 and 8 only once and neither 7 nor 8 is in the 1st place.

    ➀ Let us count the number of 6-digit positive integers containing the digit 7 and 8 only once and either 7 or 8 is in the 1st place.

    Let us assume that the digit 7 and 8 occupy the first two places.


    The 1st digit can be chosen in 2 ways, by choosing either 7 or 8.

    The 2nd digit can be chosen in just 1 way. If 7 was chosen for the 1st place, then 8 is chosen and vice-versa.

    The remaining digits can be chosen in 8 ways, that is, any digit except 7 or 8.

    Moreover there are 5 ways how to choose the place where to put 7 or 8 apart from the 1st place.

    Therefore by the multiplication principle, the number of 6-digit positive integers containing the digit 7 and 8 only once and either 7 or 8 is in the 1st place: is:

    5\times2\times 1\times8\times8\times8\times8=40960

    ➁ Let us count the number of 6-digit positive integers containing the digit 7 and 8 only once and neither 7 nor 8 is in the 1st place.

    Let us assume that the digit 7 and 8 occupy the 2nd and 3th places.


    The 1st digit can be chosen in 7 ways, by choosing any digit from 1,2,3,4,5,6,9.

    The 2nd digit can be chosen in 2 ways, by choosing either 7 or 8. The 3rd digit can be chosen in just 1 way. If 7 was chosen for the 2nd place, then 8 is chosen and vice-versa.

    The remaining digits can be chosen in 8 ways, that is, any digit except 7 or 8.

    Moreover there are \begin{pmatrix}5\\2\end{pmatrix} ways how to choose the two places where to put the 7 and 8, excluding the 1st place.

    Therefore by the multiplication principle, the number of 6-digit positive integers containing the digit 7 and 8 only once and neither 7 nor 8 is in the 1st place is:

    \begin{pmatrix}5\\2\end{pmatrix}\times 7\times2\times1\times8\times8\times8\times8=71680

    Therefore, the number of 6-digit positive integers containing the digit 7 and 8 only once is:

    40960+71680=112640

  • September 2019 Paper 1 Question 10

    (a) Let the given function be expressed as the square of a quadratic expression:

    \begin{aligned} x^4+12x^3+46x^2+px+q&\equiv (ax^2+bx+c)^2\\ &\equiv (ax^2+bx+c)(ax^2+bx+c) \end{aligned}

    Equating coefficients of x^4:

    \begin{aligned} 1&=a^2\\ \therefore a&=\pm 1 \end{aligned}

    Let us assume that a=1.

    Note that we could have chosen a to be -1 and then we would get values of \ b and c with opposite signs than the values obtained in this case. Ultimately both cases will give us the same values for p and q.

    Equating coefficients of x^3:

    \begin{aligned} 12&=ab+ba\\ 12&=2ab\\ 12&=2(1)b\text{ (since }a=1)\\ \therefore b&=6 \end{aligned}

    Equating coefficients of x^2:

    \begin{aligned} 46&=ac+b^2+ca\\ 46&=2ac+b^2\\ 46&=2(1)c+6^2\text{ (since }a=1\text{ and }b=6)\\ \therefore c&=5 \end{aligned}

    Equating coefficients of x:

    \begin{aligned} p&=bc+cb\\ p&=2bc\\ p&=2(6)(5)\text{ (since }b=6\text{ and }c=5)\\ \therefore p&=60 \end{aligned}

    Equating constants:

    \begin{aligned} q&=c^2\\ q&=5^2 \text{ (since }c=5)\\ \therefore q&=25 \end{aligned}

    Hence: p=60 and q=25.

    (b) Let us simplify the given equation to one in quadratic form:

    \frac{x}{x-a}+\frac{x}{x-b}=1+c

    Making a common denominator for the L.H.S.:

    \frac{x(x-b)+x(x-a)}{(x-a)(x-b)}=1+c

    Taking the denominator on the L.H.S. as a factor on the R.H.S.:

    x(x-b)+x(x-a)=(1+c)(x-a)(x-b)

    Expanding:

    x^2-bx+x^2-ax=(1+c)x^2-(1+c)(a+b)x+(1+c)ab

    Putting all the terms on the L.H.S.:

    (1-c)x^2+c(a+b)x-ab(1+c)=0\ \ \ \ ...(1)

    Recall that in general, the quadrating equation a'x^2+b'x+c'=0 has real and equal roots if and only if b'^2-4a'c'=0. Hence considering Equation (1):

    \begin{aligned} c^2(a+b)^2+4(1-c)(ab)(1+c)&=0\\ c^2(a+b)^2+4(1-c^2)(ab)&=0\\ c^2(a+b)^2+4ab-4c^2(ab)&=0\\ c^2[(a+b)^2-4ab]+4ab&=0\\ c^2(a-b)^2&=-4ab\text{ (note that }(a+b)^2-4ab=(a-b)^2)\\ \therefore c^2&=-\frac{4ab}{(a-b)^2}\ \square \end{aligned}

    \begin{aligned} c^2&=-\frac{4ab}{(a-b)^2}\\ &=1-1-\frac{4ab}{(a-b)^2}\\ &=1-\frac{(a-b)^2+4ab}{(a-b)^2}\\ &=1-\frac{(a+b)^2}{(a-b)^2}\text{ (since }(a+b)^2-4ab=(a-b)^2)\\ &=1-\begin{pmatrix}\frac{a+b}{a-b}\end{pmatrix}^2\ \square \end{aligned}

    Note that c is a real number, \therefore c^2\geq 0. Suppose that c^2=0:

    \begin{aligned} \therefore 1-\begin{pmatrix}\frac{a+b}{a-b}\end{pmatrix}^2&=0\\ (a+b)^2&=(a-b)^2\\ ab&=0 \end{aligned}

    \therefore a=0\text{ or }b=0

    This is not possible since both a and b should be non-zero. Hence c^2\neq 0. Hence c^2 >0.

    On the other hand, \begin{pmatrix}\frac{a+b}{a-b}\end{pmatrix}^2 is a non-negative number. Therefore the maximum of c^2=1-\begin{pmatrix}\frac{a+b}{a-b}\end{pmatrix}^2 is 1 and this is obtained when a=-b. Hence c^2\leq 1.

    \therefore 0\lt c^2\leq 1\ \square
  • September 2019 Paper 2 Question 1

    (a) Let the Integrating Factor be:

    \begin{aligned} \text{I.F.}&=e^{\int (2x+1)dx}\\ &=e^{x^2+x} \end{aligned}

    We multiply the original equation by the I.F. to get an exact differential equation:

    \begin{aligned}e^{x^2+x}\frac{dy}{dx}+(2x+1)ye^{x^2+x}-e^{-x^2}e^{x^2+x}&=0\\e^{x^2+x}\frac{dy}{dx}+(2x+1)ye^{x^2+x}-e^x&=0\end{aligned}

    Using the product rule in reverse:

    \frac{d}{dx}(e^{x^2+x}y)-e^x=0

    Therefore:
    \begin{aligned} \frac{d}{dx}(e^{x^2+x}y)&=e^x\\ \int\frac{d}{dx}(e^{x^2+x}y)dx&=\int e^x dx\\ e^{x^2+x}y&=e^x+C \end{aligned}


    (b) Consider \frac{d^2 y}{dx^2}+4\frac{dy}{dx}+4y=0

    The auxiliary equation is:

    \begin{aligned} m^2+4m+4&=0\\ (m+2)(m+2)&=0\\ m&=-2 \end{aligned}

    We have real and repeat roots. Therefore the Complimentary Function (C.F.) is:

    \begin{aligned} y=(A+Bx)e^{-2x} \end{aligned}

    Let the Particular Integral be:

    \begin{aligned} y&=ax^2+bx+c&\ \ \ \ ...(1)\\ \therefore\frac{dy}{dx}&=2ax+b &\ \ \ \ ...(2)\\ \text{and }\frac{d^2 y}{dx^2}&=2a &\ \ \ \ ...(3) \end{aligned}

    Substituting Equations (1), (2) and (3) in the original 2nd Order Differential Equation gives us:

    2a+4(2ax+b)+4(ax^2+bx+c)\equiv 8x^2

    Let us find the values of a, b and c.

    Equating the coefficients of x^2:

    \begin{aligned} 4a&=8\\ a&=2 \end{aligned}

    Equating the coefficients of x:

    \begin{aligned} 8a+4b&=0\\ 8(2)+4b&=0\text{ (since }a=2)\\ b&=-4 \end{aligned}

    Equating constants:

    \begin{aligned} 2a+4b+4c&=0\\ 2(2)+4(-4)+4c&=0\text{ (since }a=2\text{ and }b=-4)\\ c&=3 \end{aligned}

    Therefore the Particular Integral (P.I.) is:

    y=2x^2-4x+3

    The General Solution (G.S.) is:

    \begin{aligned} y&=\text{C.F.}+\text{P.I.}\\ y&=(A+Bx)e^{-2x}+2x^2-4x+3\ \ \ \ ...(4) \end{aligned}

    We will use the initial conditions to find A and B in order to find the Particular Solution (P.S.).

    Substituting x=0 and y=0 in Equation (4):

    \begin{aligned} 0&=Ae^{-2(0)}+2(0)^2-4(0)+3\\ \therefore A&=-3 \end{aligned}

    By differentiating Equation (4) we get:

    \frac{dy}{dx}=-2(A+Bx)e^{-2x}+Be^{-2x}+4x-4\ \ \ \ (5)

    Substituting x=0 and \frac{dy}{dx}=0 in Equation (5):

    \begin{aligned} 0&=-2(A+0)e^{-2(0)}+Be^{-2(0)}+4(0)-4\\ &B=-2 \end{aligned}

    By substituting A=-3 and B=-2 in the G.S. (Equation (4)), we get the Particular Solution (P.S.):

    \begin{aligned} y&=(-3-2x)e^{-2x}+2x^2-4x+3\\ y&=-(3+2x)e^{-2x}+2x^2-4x+3 \end{aligned}
  • September 2019 Paper 2 Question 2

    *****Includes Newton Rhapson Trapezium Rule and Simpson's Rule *****

    2 (a) Consider the function f(x)=\sin(2x)+\ln(x)

    \begin{aligned} f(0.2)&=\sin(2(0.2))+\ln(0.2)\\ &=\sin(0.4)+\ln(0.2)\\ &=-1.22\\ &\lt 0 \end{aligned}

    \begin{aligned} f(1)&=\sin(2(1))+\ln(1)\\ &=\sin(2)+\ln(1)\\ &=0.91\\ &\gt 0 \end{aligned}

    Since the function f changes its sign in the interval [0.2,1], it has a root in such an interval.

    Now let us use Newton-Rhapson's Method to find an approximation of this root.

    \begin{aligned} f(x)&=\sin(2x)+\ln(x)\\ f'(x)&=2\cos(2x)+\frac{1}{x} \end{aligned}

    1st iteration:

    x_1=0.6


    \begin{aligned} f(0.6)&=\sin(2(0.6))+\ln(0.6)=0.4212\\ f'(0.6)&=2\cos(2(0.6))+\frac{1}{0.6}=2.3914 \end{aligned}

    By Newton-Rhapson's Approximation:

    \begin{aligned} x_2&=x_1-\frac{f(x_1)}{f'(x_1)}\\ &=0.6-\frac{f(0.6)}{f'(0.6)}\\ &=0.6-\frac{0.4212}{2.3914}\\ &=0.4239 \end{aligned}

    2nd iteration:

    x_2=0.4239


    \begin{aligned} f(0.4239)&=\sin(2(0.4239))+\ln(0.4239)=-0.1086\\ f'(0.4239)&=2\cos(2(0.4239))+\frac{1}{0.4239}=3.6826 \end{aligned}

    By Newton-Rhapson's Approximation:

    \begin{aligned} x_3&=x_2-\frac{f(x_2)}{f'(x_2)}\\ &=0.4239-\frac{f(0.4239)}{f'(0.4239)}\\ &=0.4239-\frac{0.1086}{3.6826}\\ &=0.4533 \end{aligned}

    (b) Since the interval width h=1 we are going to consider the value of the function y=\frac{1}{\ln (x+4)} at the following x-values:

    x -1 0 1 2 3
    y=\frac{1}{\ln (x+4)} y_0=\frac{1}{\ln 3} y_1=\frac{1}{\ln 4} y_2=\frac{1}{\ln 5} y_3=\frac{1}{\ln 6} y_4=\frac{1}{\ln 7}
    (i) Using the Trapezoidal Rule with n=4:

    \begin{aligned} \int_{-1}^{3}\frac{1}{\ln(x+4)}dx&\simeq \frac{1}{2}h \{(y_0+y_4)+2(y_1+y_2+y_3)\}\\ &=\frac{1}{2}(1) \begin{Bmatrix}(\frac{1}{\ln 3}+\frac{1}{\ln 7})+2(\frac{1}{\ln 4}+\frac{1}{\ln 5}+\frac{1}{\ln 6})\end{Bmatrix}\text{ (since }h=1)\\ &=2.613 \end{aligned}
    (ii) Using Simpson's Rule with n=4:

    \begin{aligned} \int_{-1}^{3}\frac{1}{\ln(x+4)}dx&\simeq \frac{1}{3}h \{(y_0+y_4)+4(y_1+y_3)+2(y_2)\}\\ &=\frac{1}{3}(1) \begin{Bmatrix}(\frac{1}{\ln 3}+\frac{1}{\ln 7})+4(\frac{1}{\ln 4}+\frac{1}{\ln 6})+2(\frac{1}{\ln 5})\end{Bmatrix}\text{ (since }h=1)\\ &=2.595 \end{aligned}

    The error for the Trapezoidal Rule estimation is:

    \text{Exact}-\text{Estimate}=2.593-2.613=-0.02

    The error for the Simpson's Rule estimation is:

    \text{Exact}-\text{Estimate}=2.593-2.595=-0.002

    The Simpson's Rule Estimation gives a lower absolute error, therefore it offers a better estimate.
  • September 2019 Paper 2 Question 5

    (a) Proof by Induction.

    Induction Basis: Let n=8.

    \begin{aligned} \text{L.H.S.}&=n^2=8^2=64\\ \text{R.H.S.}&=7k+1=7(8)+1=57 \end{aligned}

    \therefore\text{L.H.S.}\gt\text{R.H.S.}
    \therefore Result holds for n=8.

    Induction Hypothesis: Assume that the result holds for n=k, that is, k^2 \gt 7k+1.

    Induction Step: Let us show that the result holds for n=k+1.

    \begin{aligned} \text{L.H.S.}&=n^2\\ &=(k+1)^2\\ &=k^2+2k+1\\ &> (7k+1) +2k+1\text{ (by the Induction Hypothesis)}\\ &=9k+2\\ &=7k+7+2k-5\\ &=7(k+1)+2k-5\\ &> 7(k+1)+1 \text{ (note that since }k\geq 8\text{, then it follows that }2k-5>1)\\ &=\text{R.H.S.} \end{aligned}

    \therefore\text{L.H.S.}\gt\text{R.H.S.}
    \therefore Result holds for n=k+1.

    \therefore Result holds by Induction for all n\geq 8. \square

    (b) (i) Let n=1 in the recurrence relation u_{n+1}=\frac{2 u_n-1}{3}:

    \begin{aligned} u_2&=\frac{2u_1-1}{3}\\ &=\frac{2(1)-1}{3}\text{ (since }u_1=1)\\ &=\frac{1}{3} \end{aligned}

    Let n=2 in the recurrence relation:

    \begin{aligned} u_3&=\frac{2u_2-1}{3}\\ &=\frac{2(\frac{1}{3})-1}{3}\text{ (since }u_2=\frac{1}{3})\\ &=-\frac{1}{9} \end{aligned}

    Let n=3 in the recurrence relation:

    \begin{aligned} u_4&=\frac{2u_3-1}{3}\\ &=\frac{2(-\frac{1}{9})-1}{3}\text{ (since }u_3=-\frac{1}{9})\\ &=-\frac{11}{27} \end{aligned}

    Let n=4 in the recurrence relation:

    \begin{aligned} u_5&=\frac{2u_4-1}{3}\\ &=\frac{2(-\frac{11}{27})-1}{3}\text{ (since }u_4=-\frac{11}{27})\\ &=-\frac{49}{81} \end{aligned}

    Hence the first 5 terms of the sequence are: 1,\frac{1}{3},-\frac{1}{9},-\frac{11}{27},-\frac{49}{81}.

    (ii) Consider the equation u_n=3(\frac{2}{3})^n-1.

    Induction Basis: Let n=1.

    \begin{aligned} \text{L.H.S.}&=u_1\\ &=1\text{ (from Part (b) (i))} \end{aligned}

    \begin{aligned}\text{R.H.S.}&=3\begin{pmatrix}\frac{2}{3}\end{pmatrix}^1-1\\&=1 \end{aligned}

    \therefore\text{L.H.S.}=\text{R.H.S.}
    \therefore Result holds for n=1.

    Induction Hypothesis: Assume that the result holds for n=k, that is, u_k=3(\frac{2}{3})^k-1.

    Induction Step: Let us show that the result holds for n=k+1.

    \begin{aligned} \text{L.H.S.}&=u_{k+1}\\ &=\frac{2u_k-1}{3}\text{ using the recurrence relation given in Part (b) (i))}\\ &=\frac{2(3(\frac{2}{3})^k-1)-1}{3}\text{ (by the Induction Hypothesis)}\\ &=\frac{2(3)(\frac{2}{3})^k-2-1}{3}\\ &=\frac{2(3)(\frac{2}{3})^k-3}{3}\\ &=2\begin{pmatrix}\frac{3}{3}\end{pmatrix}\begin{pmatrix}\frac{2}{3}\end{pmatrix}^k-1\\ &=3\begin{pmatrix}\frac{2}{3}\end{pmatrix}^{k+1}-1\\ &=\text{R.H.S.} \end{aligned}

    \therefore \text{L.H.S.}= \text{R.H.S.}
    \therefore Result holds for n=k+1.

    \therefore Result holds by Induction for all n\geq 1. \square

  • September 2020 Paper 2 Question 7

    **** NOTE this question is a combination of Maclaurin's Series and Polar Coordinates **** (a) (i) Maclaurin's Theorem states that:

    f(x)=f(0)+xf'(0)+\frac{x^2}{2!}f''(0)+\frac{x^3}{3!}f'''(0)+\frac{x^4}{4!}f^{iv}(0)+\cdots\ \ \ \ ...(1)


    \begin{aligned} f(x)&=(1+e^{-x})^2\\ f'(x)&=2(1+e^{-x})(-e^{-x})\\ &=-2e^{-x}(1+e^{-x})\\ &=-2e^{-x}-2e^{-2x}\\ f''(x)&=2e^{-x}+4e^{-2x}\\ f'''(x)&=-2e^{-x}-8e^{-2x}\\ f^{iv}(x)&=2e^{-x}+16e^{-2x} \end{aligned} \begin{aligned} \qquad \end{aligned} \begin{aligned} f(0)&=(1+e^{-0})^2=4\\ \\ \\ f'(0)&=-2-2=-4\\ f''(0)&=2+4=6\\ f'''(0)&=-2-8=-10\\ f^{iv}(0)&=2+16=18 \end{aligned}

    Substituting the values for f'(0), f''(0), f'''(0) and f^{iv}(0) in the Maclaurin's expansion, given by Equation (1) results in:

    \begin{aligned} (1+e^{-x})^2&=4+x(-4)+\frac{x^2}{2!}(6)+\frac{x^3}{3!}(-10)+\frac{x^4}{4!}(18)+\cdots\\ &=4-4x+3x^2-\frac{5}{3}x^3+\frac{3}{4}x^4+\cdots \end{aligned}

    (ii) Consider the sequence: f'(0), f''(0), f'''(0),f^{iv}(0),\cdots

    In Part (a) (i) it was shown that this sequence is given by:

    -2-2, 2+4,-2-8,2+16,\cdots

    The general term of such as sequence is:

    \begin{aligned} f^{(n)}(0)&=2(-1)^n+2^n(-1)^n\\ &=(2+2^n)(-1)^n \end{aligned}

    Therefore, for n\geq 1:

    \begin{aligned} \text{The coefficient of }x^n&=\frac{f^{(n)}(0)}{n!}\\ &=\frac{(2+2^n)(-1)^n}{n!} \end{aligned}

    (b) (i) To convert an equation from Cartesian to Polar form, we use the equations:

    \begin{aligned} x^2+y^2&=r^2&\ \ \ \ ...(2)\\ x&=r\cos\theta&\ \ \ \ ...(3)\\ \text{and }y&=r\sin\theta&\ \ \ \ ...(4)\end{aligned}

    \begin{aligned} (x^2+y^2)^{\frac{3}{2}}&=x^2+5y^2\\ (r^2)^{\frac{3}{2}}&=r^2\cos^2\theta+5r^2\sin^2\theta\text{ (using Equations (2), (3) and (4))}\\ r^3&=r^2(cos^2\theta+5\sin^2\theta)\\ r&=\cos^2\theta+5\sin^2\theta\\ r&=\cos^2\theta+5(1-\cos^2\theta)\text{ (since }\sin^2 A +\cos^2 A=1)\\ r&=5-4\cos^2\theta\\ r&=5-4\begin{pmatrix}\frac{\cos 2\theta+1}{2}\end{pmatrix}\text{ (since }\cos 2A =2\cos^2A-1)\\ r&=5-2\cos 2\theta-2\\ r&=3-2\cos 2\theta\ \square \end{aligned}

    (ii) Let us find the points where the curve touches the pole. Such points have their r-value equal to 0. Hence let r=0.

    \begin{aligned} r&=0\\ 3-2\cos 2\theta&=0\\ 2\cos 2\theta&=3\\ \cos 2\theta&=1.5 \end{aligned}

    This equation has no solution for \theta since the range of the cosine function is [1,1] and 1.5 is outside this range.

    Hence there are no points with r=0.

    Hence the curve does not touch the pole. (Note: You will visualise this in part (iii) below)

    Therefore there are no tangents to the curve at the pole. \square

    (iii)
    \theta 0 \frac{\pi}{6} \frac{\pi}{3} \frac{\pi}{2} \frac{2\pi}{3} \frac{5\pi}{6} \pi \frac{7\pi}{6} \frac{4\pi}{3} \frac{3\pi}{2} \frac{5\pi}{3} \frac{11\pi}{6} 2\pi
    r=3-2\cos2\theta 1 2 4 5 4 2 1 2 4 5 4 2 1


    (iv) Let A be the area enclosed by the curve between 0 and \frac{\pi}{2}.

    Then A=\frac{11\pi}{4}, since it is one quarter of the whole area enclosed by the curve which is given to be 11\pi.

    By using the formula for the area enclosed by a polar curve:

    \begin{aligned} A&=\frac{1}{2}\int_{0}^{\frac{\pi}{2}}r^2 d\theta\\ A&=\frac{1}{2}\int_{0}^{\frac{\pi}{2}}(3-2\cos 2\theta)^2 d\theta\\ \frac{11\pi}{4}&=\frac{1}{2}\int_{0}^{\frac{\pi}{2}}(3-2\cos 2\theta)^2 d\theta\\ \therefore\int_{0}^{\frac{\pi}{2}}(3-2\cos 2\theta)^2 d\theta &=\frac{11\pi}{2} \end{aligned}
  • September 2019 Paper 2 Question 10

    (a)
    \begin{aligned} &\mathbb{P}[\text{\textquotedblleft Alarm 1 fails\textquotedblright}\cap\text{\textquotedblleft Alarm 2 fails\textquotedblright}\cap\text{\textquotedblleft Alarm 3 fails\textquotedblright}]\\ =&\mathbb{P}[\text{\textquotedblleft Alarm 1 fails\textquotedblright}]\mathbb{P}[\text{\textquotedblleft Alarm 2 fails\textquotedblright}]\mathbb{P}[\text{\textquotedblleft Alarm 3 fails\textquotedblright}]\\ &\ \ \ \ \ \ \ \ \text{ (by independence: }\mathbb{P}[A\cap B]=\mathbb{P}[A]\mathbb{P}[B])\\ =&(0.01)(0.01)(0.01) \end{aligned}

    (b)
    \begin{aligned} \mathbb{P}[\text{\textquotedblleft 1 Black ball\textquotedblright}]&=\mathbb{P}[1^{st}\text{ selection is White}\cap 2^{nd}\text{ selection is White}]\\ &=\mathbb{P}[1^{st}\text{ sel. is White}]\mathbb{P}[2^{nd}\text{ sel. is White}|1^{st}\text{ sel. is White}]\\ &\ \ \ \ \ \ \ \ \text{ (since: }\mathbb{P}[A|B]=\frac{\mathbb{P}[A\cap B]}{\mathbb{P}[B]})\\ &=\begin{pmatrix}\frac{1}{2}\end{pmatrix}\begin{pmatrix}\frac{2}{3}\end{pmatrix}\\ &=\frac{1}{3} \end{aligned}

    \begin{aligned} \mathbb{P}[\text{\textquotedblleft 2 Black balls\textquotedblright}]&=\mathbb{P}[1^{st}\text{ selection is Black}\cap 2^{nd}\text{ selection is White}]\\ &\ \ +\mathbb{P}[1^{st}\text{ selection is White}\cap 2^{nd}\text{ selection is Black}]\\ &=\mathbb{P}[1^{st}\text{ sel. is Black}]\mathbb{P}[2^{nd}\text{ sel. is White}|1^{st}\text{ sel. is Black}]\\ &\ \ +\mathbb{P}[1^{st}\text{ sel. is White}]\mathbb{P}[2^{nd}\text{ sel. is Black}|1^{st}\text{ sel. is White}]\\ &=\begin{pmatrix}\frac{1}{2}\end{pmatrix}\begin{pmatrix}\frac{1}{3}\end{pmatrix}+\begin{pmatrix}\frac{1}{2}\end{pmatrix}\begin{pmatrix}\frac{1}{3}\end{pmatrix}\\ &=\frac{1}{3} \end{aligned}

    \begin{aligned} \mathbb{P}[\text{\textquotedblleft 3 Black balls\textquotedblright}]&=\mathbb{P}[1^{st}\text{ selection is Black}\cap 2^{nd}\text{ selection is Black}]\\ &=\mathbb{P}[1^{st}\text{ sel. is Black}]\mathbb{P}[2^{nd}\text{ sel. is Black}|1^{st}\text{ sel. is Black}]\\ &=\begin{pmatrix}\frac{1}{2}\end{pmatrix}\begin{pmatrix}\frac{2}{3}\end{pmatrix}\\ &=\frac{1}{3} \end{aligned}

    (c)
    \begin{aligned} \mathbb{P}[\text{Biased Coin}|H]=\frac{wwww}{www} \end{aligned}

    To Continue
  • September 2020 Paper 1 Question 1a

    This is the solution of the question mentioned above. This is the solution of the question mentioned above. This is the solution of the question mentioned above.

    \(x^2/2=4\)
  • September 2020 Paper 2 Question 4

    Here it is.

  • May 2019 Paper 2 Question 10